Fourierreihe berechnen

Neue Frage »

lonesome-dreamer Auf diesen Beitrag antworten »
Fourierreihe berechnen
Hallo zusammen,
zu der Funktion soll die Fourierreihe berechnet werden.
Ich habe zunächst die Funktion umgeschrieben:

Die Periode ist nun

Dann habe ich den Koeffizienten berechnet:


Dann die Koeffizienten :


Aber wie integriere ich das jetzt?
Oder gibt es vielleicht noch einen anderen Lösungsweg?

Besten Dank schon mal im Voraus für eure Hilfe

Gruß
Natalie
Calvin Auf diesen Beitrag antworten »

Löse die Klammer auf und wende ein Additionstheorem an:

lonesome-dreamer Auf diesen Beitrag antworten »

Hi,
das wird ja ziemlich aufwendig, wenn man das dann noch integrieren soll.
Die Lösung ist übrigens , also genau das, wie ich das umgeschrieben habe.
Ist das Zufall, oder gibt es da einen Zusammenhang?

Gruß
Natalie
Calvin Auf diesen Beitrag antworten »

Nein, das ist kein Zufall. Deine Umformung der Funktion mit Additionstheoremen hat sie tatsächlich in eine Fourierreihe überführt. Mit ein bißchen Übung (die ich definitiv nicht habe Augenzwinkern ) hätte man das sogar sehen können.

Die sind alle 0. Das wirst du auch feststellen, wenn du das Integral ausrechnest Augenzwinkern

Die sind fast alle 0. Lediglich und sind nicht 0. Auch das könntest du über ein Integral ausrechnen.
WebFritzi Auf diesen Beitrag antworten »

Zitat:
Original von Calvin
Die sind alle 0. Das wirst du auch feststellen, wenn du das Integral ausrechnest Augenzwinkern


Das Integral braucht man dafür nicht auszurechnen, denn ungerade mal gerade ergibt ungerade. Augenzwinkern
lonesome-dreamer Auf diesen Beitrag antworten »

Danke schon mal.

Ich hab jetzt noch ne andere Aufgabe, bei der ich mich schwer tue.
im Bereich und Periode .

Ich habe für ausgerechnet.

Und jetzt wieder die Frage nach den Koeffizienten .
.

Stimmt das bis dahin? Und wenn ja, wie integriert man das am besten, oder gibt's noch ne einfachere Möglichkeit, die Aufgabe zu lösen?
 
 
WebFritzi Auf diesen Beitrag antworten »

f(x) ist eine gerade Funktion. Was kannst du daraus alles schließen?
lonesome-dreamer Auf diesen Beitrag antworten »

Das bedeutet doch, dass für alle n ist, oder?
WebFritzi Auf diesen Beitrag antworten »

Ja. Und dass der Integrand von a_n eine gerade Funktion ist. Das heißt, du brauchst nicht beide Integrale zu berechnen, sondern musst nur das erste (oder das zweite) mit 2 multiplizieren.
lonesome-dreamer Auf diesen Beitrag antworten »

also so:
?
Und wie integriert man das am besten? Partiell?
WebFritzi Auf diesen Beitrag antworten »

Warum versuchst du es nicht einfach mal?
lonesome-dreamer Auf diesen Beitrag antworten »

Wie berücksichtige ich denn die Integrationsgrenzen bei der partiellen Integration?
WebFritzi Auf diesen Beitrag antworten »

lonesome-dreamer Auf diesen Beitrag antworten »

Puh. Das ist ja wirklich nicht schön.
Das muss doch auch noch anders gehen.
Jedenfalls komme ich auf:
WebFritzi Auf diesen Beitrag antworten »



lonesome-dreamer Auf diesen Beitrag antworten »

für gerade n ist und für ungerade n ist
und für alle n ist .
Das heißt der komplette rechte Term fällt weg und es bleibt übrig:


Stimmt das so?
WebFritzi Auf diesen Beitrag antworten »

Zitat:
Original von lonesome-dreamer
für gerade n ist und für ungerade n ist
und für alle n ist .
Das heißt der komplette rechte Term fällt weg und es bleibt übrig:


Stimmt das so?


Das stimmt alles. Jetzt überleg dir mal, was cos²(n pi) ist.

P.S.: Bist du sicher, dass da cos²(n pi) steht? Mein Mathematica sagt mir, dass da nur cos(n pi) steht.
lonesome-dreamer Auf diesen Beitrag antworten »

Zitat:
Original von WebFritzi
P.S.: Bist du sicher, dass da cos²(n pi) steht? Mein Mathematica sagt mir, dass da nur cos(n pi) steht.

O ja. Mathematica hat natürlich recht Augenzwinkern
Ich hab mich vertippt. Es soll nur sein.
Also so:


Aber wie gehts jetzt weiter?
Je nachdem ob n gerade oder ungerade ist, hat der Term ein anderes Vorzeichen.
Frooke Auf diesen Beitrag antworten »

Dann setze doch
lonesome-dreamer Auf diesen Beitrag antworten »

aha.
Da hätte ich auch selbst drauf kommen können Finger1

Und das multipliziere ich jetzt noch mit und erhalte als Fourierreihe:


Richtig?
WebFritzi Auf diesen Beitrag antworten »

Du hast doch nur deine Werte in eine Formel eingesetzt. Dann wird es wohl richtig sein. Augenzwinkern
lonesome-dreamer Auf diesen Beitrag antworten »

Zitat:
Original von WebFritzi
Du hast doch nur deine Werte in eine Formel eingesetzt. Dann wird es wohl richtig sein. Augenzwinkern


Naja, ab und zu kann man selbst da Fehler machen Augenzwinkern
Jedenfalls vielen Dank euch allen für die Hilfe. Ihr seid die Besten hier Freude

Gruß
Natalie
Neue Frage »
Antworten »



Verwandte Themen

Die Beliebtesten »
Die Größten »
Die Neuesten »